Difference between revisions of "2021 AIME II Problems/Problem 7"

(Problem)
Line 1: Line 1:
 
==Problem==
 
==Problem==
These problems will not be posted until the 2021 AIME II is released on Thursday, March 25, 2021.
+
Let <math>a, b, c,</math> and <math>d</math> be real numbers that satisfy the system of equations
 +
<cmath>a + b = -3</cmath><cmath>ab + bc + ca = -4</cmath><cmath>abc + bcd + cda + dab = 14</cmath><cmath>abcd = 30.</cmath>There exist relatively prime positive integers <math>m</math> and <math>n</math> such that
 +
<cmath>a^2 + b^2 + c^2 + d^2 = \frac{m}{n}.</cmath>Find <math>m + n</math>.
 +
 
 
==Solution==
 
==Solution==
 
We can't have a solution without a problem.
 
We can't have a solution without a problem.

Revision as of 15:33, 22 March 2021

Problem

Let $a, b, c,$ and $d$ be real numbers that satisfy the system of equations \[a + b = -3\]\[ab + bc + ca = -4\]\[abc + bcd + cda + dab = 14\]\[abcd = 30.\]There exist relatively prime positive integers $m$ and $n$ such that \[a^2 + b^2 + c^2 + d^2 = \frac{m}{n}.\]Find $m + n$.

Solution

We can't have a solution without a problem.

See also

2021 AIME II (ProblemsAnswer KeyResources)
Preceded by
Problem 6
Followed by
Problem 8
1 2 3 4 5 6 7 8 9 10 11 12 13 14 15
All AIME Problems and Solutions

The problems on this page are copyrighted by the Mathematical Association of America's American Mathematics Competitions. AMC logo.png